Convergence of implicit Euler method

Click For Summary
The discussion focuses on the convergence of the implicit Euler method, specifically analyzing its local truncation error, which is determined to be ln = (-h2/2)y''(xn-1) + O(h3). The user is attempting to establish convergence using the Lipschitz condition and the triangle inequality, leading to the inequality ||en|| <= 1/(1-hL)||en-1|| - Mh2/(2(1-hL)). They express difficulty in progressing further from this point, particularly in applying the initial condition eo = y(x0) - y0 = 0. The conversation highlights the mathematical intricacies involved in proving convergence for numerical methods.
squenshl
Messages
468
Reaction score
4

Homework Statement


The implicit Euler method is yn = yn-1 + hf(xn,yn).
Find the local truncation error and hence show that the method is convergent.


Homework Equations





The Attempt at a Solution


I found the error to be ln = (-h2/2)y''(xn-1) + O(h3).
For convergence I am up to using the Lipschitz condition, triangle inequality and ||ln|| = -Mh2/2 to get:
||en|| <= 1/(1-hL)||en-1|| - Mh2/(2(1-hL)) for hL <= 1/2 but I am stuck after this. Someone help please.
 
Physics news on Phys.org
Since eo = y(x0) - y0 = 0
||en|| <= -Mh2/(2(1-hL))(1+(1-hL)+...+(1-hL)n-1) = -Mh2/(2(1-hL))*((1-hL)n-1 - 1)/hL
 
Question: A clock's minute hand has length 4 and its hour hand has length 3. What is the distance between the tips at the moment when it is increasing most rapidly?(Putnam Exam Question) Answer: Making assumption that both the hands moves at constant angular velocities, the answer is ## \sqrt{7} .## But don't you think this assumption is somewhat doubtful and wrong?

Similar threads

  • · Replies 12 ·
Replies
12
Views
2K
Replies
2
Views
2K
  • · Replies 1 ·
Replies
1
Views
2K
  • · Replies 1 ·
Replies
1
Views
3K
Replies
6
Views
2K
  • · Replies 2 ·
Replies
2
Views
5K
Replies
4
Views
4K
  • · Replies 2 ·
Replies
2
Views
2K
Replies
4
Views
2K
  • · Replies 10 ·
Replies
10
Views
5K